{{tag>electrostatic field_lines exam_ee2_SS2022}}{{include_n>1000}} #@TaskTitle_HTML@##@Lvl_HTML@#~~#@ee1_taskctr#~~ Electrostatics I\\ (written test, approx. 10 % of a 120-minute written test, SS2022) #@TaskText_HTML@# Given is the arrangement of electric charges in the picture below. The values of the point charges are * $q_0=-1 ~\rm nC$ * $q_1=-2 ~\rm nC$ * $q_2=q_3=+5 ~\rm nC$ In the beginning, the area charge is $q_4=0 ~\rm nC$. \\ The permittivity is $\varepsilon_{\rm r} \varepsilon_0 = 8.854 \cdot 10^{-12} ~\rm As/Vm$ {{drawio>ee2:dtoqvpvrbdtcozfk_question1.svg}} 1. Calculate the single forces $\vec{F}_{01}$, $\vec{F}_{02}$, $\vec{F}_{03}$, on the charge $q_0$! #@HiddenBegin_HTML~dtoqvpvrbdtcozfk_11,Path~@# First, calculate the magnitude of the forces, like $\vec{F}_{01}$. \\ The force $\vec{F}_{01}$ is purely on the $x$-axis and therefore equal to $F_{01,x}$. \begin{align*} \vec{F}_{01} = F_{01,x} &= {{1}\over{4\pi\varepsilon}}\cdot {{q_1\cdot q_0}\over{r^2_{01}}} \\ &= {{1}\over{4\pi\cdot 8.854 \cdot 10^{-12} ~\rm As/Vm}}\cdot {{1 \cdot 10^{-9} ~\rm C \cdot 2 \cdot 10^{-9} ~\rm C}\over{(3 \cdot 10^{-2} ~\rm m)^2}} \\ &= 19.97... \cdot 10^{-6} {\rm {{(As)^2 \cdot Vm}\over{As \cdot m^2}}} = 19.97... \cdot 10^{-6} {\rm {{VAs}\over{m}}} = 19.97... \cdot 10^{-6} {\rm {{Ws}\over{m}}} \\ &= 19.97... {\rm \mu N} \quad \text{(to the right)} \end{align*} Similarly, we get for $\vec{F}_{02}$ and $\vec{F}_{03}$ \begin{align*} \vec{F}_{02} = F_{02,x} &= -28.09... {\rm \mu N} \quad \text{(to the right)} \\ \vec{F}_{03} &= -22.47... {\rm \mu N} \quad \text{(to the top left)} \\ \end{align*} For $\vec{F}_{03}$, we have to calculate the $x$- and $y$-component. \\ This is possible, by using the angle $\alpha$ between the line through $q_0$ and $q_3$ and the positive $x$-axis (pointing to the right). \\ So, $\alpha$ has to be between $90°$ and $180°$. It can be calculated by: \begin{align*} \alpha = \arctan(\rm {{-4~cm}\over{+2~cm}}) = \pi - 1.1071... = 180° - 63.4...° = 116.6...° \end{align*} Based on this, the $x$- and $y$-component is: \begin{align*} F_{03,x} &= |\vec{F}_{03}| \cdot \cos \alpha = 10.05... {~\rm \mu N} \text{(to the left)} \\ F_{03,y} &= |\vec{F}_{03}| \cdot \sin \alpha = 20.10... {~\rm \mu N} \text{(to the top)} \\ \end{align*} #@HiddenEnd_HTML~dtoqvpvrbdtcozfk_11,Path ~@# #@HiddenBegin_HTML~dtoqvpvrbdtcozfk_12,Result~@# * $\vec{F}_{01} = \left( {\begin{array}{cccc} 19.97 {~\rm \mu N} \\ 0 \\ \end{array} } \right)$ * $\vec{F}_{02} = \left( {\begin{array}{cccc} 28.09 {~\rm \mu N} \\ 0 \\ \end{array} } \right)$ * $\vec{F}_{03} = \left( {\begin{array}{cccc} -10.05 {~\rm \mu N} \\ 20.10 {~\rm \mu N} \\ \end{array} } \right)$ #@HiddenEnd_HTML~dtoqvpvrbdtcozfk_12,Result~@# 2. What is the magnitude of the resulting force? #@HiddenBegin_HTML~dtoqvpvrbdtcozfk_21,Path~@# With all the $x$- and $y$-components, we can calculate the resulting magnitude of the force with the Pythagorean Theorem: \\ \begin{align*} F = |\vec{F}| &= \sqrt{\left( \sum_i F_{i,x} \right)^2 + \left( \sum_i F_{i,y} \right)^2} \\ &= \sqrt{\left( 19.97 {~\rm \mu N} + 28.09 {~\rm \mu N} -10.05 {~\rm \mu N} \right)^2 + \left( 20.10 {~\rm \mu N} \right)^2} \\ &= 42.99... {~\rm \mu N} \\ \end{align*} #@HiddenEnd_HTML~dtoqvpvrbdtcozfk_21,Path ~@# #@HiddenBegin_HTML~dtoqvpvrbdtcozfk_22,Result~@# $F = 43.0 {~\rm \mu N} $ #@HiddenEnd_HTML~dtoqvpvrbdtcozfk_22,Result~@# 3. Now the charges $q_2=q_3$ are set to 0. The area charge $q_4$ generates a homogenous field of $E_4$. Which value needs $E_4$ to have to get a resulting force of $0 ~\rm N$ on $q_0$? #@HiddenBegin_HTML~dtoqvpvrbdtcozfk_31,Path~@# In the homogenous field the force is calculated by $F = E \cdot q$. \\ Here, this field has to compensate for the force $\vec{F}_{01}$ from $q_1$ on $q_0$: \begin{align*} |\vec{F}_{01}| &= |E_4| \cdot |q_0| \\ \rightarrow E_4 &= {{|\vec{F}_{01}|}\over{|q_0|}} \\ &= {{19.97... \cdot 10^{-6} {~\rm N}}\over{1 \cdot 10^{-9} ~\rm C }} \\ &= 19.97... \cdot 10^{3}{{\rm N}\over{\rm C}} \\ &= 19.97... \cdot 10^{3}{{\rm VAs/m}\over{\rm As}} \\ &= 19.97... \cdot 10^{3}{{\rm V}\over{\rm m}} \\ \end{align*} #@HiddenEnd_HTML~dtoqvpvrbdtcozfk_31,Path ~@# #@HiddenBegin_HTML~dtoqvpvrbdtcozfk_32,Result~@# $E_4 = 19.97 {{\rm kV}\over{\rm m}}$ #@HiddenEnd_HTML~dtoqvpvrbdtcozfk_32,Result~@# #@TaskEnd_HTML@#